Last visit was: 24 Apr 2024, 03:03 It is currently 24 Apr 2024, 03:03

Close
GMAT Club Daily Prep
Thank you for using the timer - this advanced tool can estimate your performance and suggest more practice questions. We have subscribed you to Daily Prep Questions via email.

Customized
for You

we will pick new questions that match your level based on your Timer History

Track
Your Progress

every week, we’ll send you an estimated GMAT score based on your performance

Practice
Pays

we will pick new questions that match your level based on your Timer History
Not interested in getting valuable practice questions and articles delivered to your email? No problem, unsubscribe here.
Close
Request Expert Reply
Confirm Cancel
SORT BY:
Date
Tags:
Show Tags
Hide Tags
Math Expert
Joined: 02 Sep 2009
Posts: 92901
Own Kudos [?]: 618689 [6]
Given Kudos: 81586
Send PM
avatar
Manager
Manager
Joined: 11 Sep 2013
Posts: 96
Own Kudos [?]: 259 [1]
Given Kudos: 26
Send PM
GMAT Club Legend
GMAT Club Legend
Joined: 08 Jul 2010
Status:GMAT/GRE Tutor l Admission Consultant l On-Demand Course creator
Posts: 5957
Own Kudos [?]: 13386 [2]
Given Kudos: 124
Location: India
GMAT: QUANT+DI EXPERT
Schools: IIM (A) ISB '24
GMAT 1: 750 Q51 V41
WE:Education (Education)
Send PM
User avatar
Intern
Intern
Joined: 10 Oct 2013
Posts: 33
Own Kudos [?]: 25 [1]
Given Kudos: 39
Location: India
Concentration: Technology, Entrepreneurship
GMAT 1: 700 Q49 V35
GMAT 2: 750 Q51 V40
GPA: 3.85
WE:Research (Telecommunications)
Send PM
Re: Is a + a^(-1) > 2? (1) a > 0 (2) a < 1 [#permalink]
1
Kudos
Is \(a + \frac{1}{a} -2 > 0\)
\((\sqrt{a} - \frac{1}{\sqrt{a}}) ^ 2 > 0\)

The above equation fails if we have a = 1. So, B is necessary.
If a < 0 \(\sqrt{a}\) is a complex number. I don't know if GMAT allows that.

So,
Tutor
Joined: 16 Oct 2010
Posts: 14816
Own Kudos [?]: 64889 [1]
Given Kudos: 426
Location: Pune, India
Send PM
Re: Is a + a^(-1) > 2? (1) a > 0 (2) a < 1 [#permalink]
1
Kudos
Expert Reply
Bunuel wrote:
Is a + a^(-1) > 2?

(1) a > 0
(2) a < 1

Kudos for a correct solution.


Given expression: \(a + \frac{1}{a}\)

If a is negative, the expression becomes negative and hence less than 2.

Assuming a is positive,
Note that the product of the numbers is a*(1/a) = 1. Given a constant product, the sum of two positive numbers is minimum when they are equal. When a = 1/a, a^2 = 1 and hence a = 1
When a = 1, a + 1/a = 2
So minimum value of a + 1/a is 2. For all other positive values of a, the sum will be more than 2.
Both statements together tell us that a is positive and not 1. So in every case, the sum a + 1/a will be greater than 2. Together they are sufficient to answer the question with "Yes".

Answer (C)
GMAT Tutor
Joined: 24 Jun 2008
Posts: 4128
Own Kudos [?]: 9241 [1]
Given Kudos: 91
 Q51  V47
Send PM
Re: Is a + a^(-1) > 2? (1) a > 0 (2) a < 1 [#permalink]
1
Kudos
Expert Reply
Statement 2 is not sufficient, since the answer is 'no' for negative values of a, and 'yes' for all positive values except a=1, as we'll see below.

When, using Statement 1 alone, we know a > 0, we can rephrase the question by multiplying by a on both sides; since a > 0, we don't need to be concerned about whether to reverse the inequality:

Is a + a^(-1) > 2 ?
Is a^2 + 1 > 2a ?
Is a^2 - 2a + 1 > 0 ?
Is (a - 1)^2 > 0 ?

So we've rephrased our question, and it's now easier to see which values of a will make the answer to our question 'yes'. We have a square on the left side of the inequality, so the left side is always positive, except when it is exactly equal to 0. It is only exactly 0 when a = 1, so the answer to our question is 'yes' except if a=1. So we need Statement 2 as well, to rule out the possibility that a=1, and the answer is C.
avatar
Intern
Intern
Joined: 29 Mar 2015
Posts: 39
Own Kudos [?]: 51 [1]
Given Kudos: 9
Location: United States
Send PM
Re: Is a + a^(-1) > 2? (1) a > 0 (2) a < 1 [#permalink]
1
Kudos
Bunuel wrote:
Is a + a^(-1) > 2?

(1) a > 0
(2) a < 1

Kudos for a correct solution.



1: Insufficient. If a = 1, a+1/a = 2. Otherwise, for all other values of a, a+1/a > 2.
2: Insufficient. If 0 < a < 1, a+1/a > 2. If a<0, a+1/a<0<2.
Together it is sufficient. Answer is C.
Math Expert
Joined: 02 Sep 2009
Posts: 92901
Own Kudos [?]: 618689 [0]
Given Kudos: 81586
Send PM
Re: Is a + a^(-1) > 2? (1) a > 0 (2) a < 1 [#permalink]
Expert Reply
Bunuel wrote:
Is a + a^(-1) > 2?

(1) a > 0
(2) a < 1

Kudos for a correct solution.


MANHATTAN GMAT OFFICIAL SOLUTION:

Because there are no constraints on a, we should consider the Standard Number Testing Set of numbers. The inequality can be rephrased as follows:
Is a + a^(-1) > 2? --> Is a + 1/a > 2?

Combining the two constraints, we have 0 < a < 1. Within the Standard Number Testing Set, only 1/2 is within this range. Also, because the answer to the question is “No” at a = 1 and “Undefined”at a = 0, we should apply the boundary principle—test numbers close to the boundaries of the range.



It seems that at the extreme edges, the values are greater than 2, so Statements (1) & (2) appear to be SUFFICIENT.

The correct answer is C.


Attachment:
2015-06-22_1645.png
2015-06-22_1645.png [ 121.84 KiB | Viewed 3821 times ]

Attachment:
2015-06-22_1646.png
2015-06-22_1646.png [ 42.47 KiB | Viewed 3829 times ]
Manager
Manager
Joined: 14 Jul 2014
Posts: 126
Own Kudos [?]: 49 [0]
Given Kudos: 110
Location: United States
Schools: Duke '20 (D)
GMAT 1: 720 Q50 V37
GMAT 2: 600 Q48 V27
GPA: 3.2
Send PM
Re: Is a + a^(-1) > 2? (1) a > 0 (2) a < 1 [#permalink]
Ahhh! Missed out on a = 1, presumed a = 1 satisfies the question and picked A.
Math Revolution GMAT Instructor
Joined: 16 Aug 2015
Posts: 10161
Own Kudos [?]: 16593 [0]
Given Kudos: 4
GMAT 1: 760 Q51 V42
GPA: 3.82
Send PM
Re: Is a + a^(-1) > 2? (1) a > 0 (2) a < 1 [#permalink]
Expert Reply
Forget conventional ways of solving math questions. In DS, Variable approach is the easiest and quickest way to find the answer without actually solving the problem. Remember equal number of variables and independent equations ensures a solution.

Is a + a^(-1) > 2?

(1) a > 0
(2) a < 1

We get a^3+a>2a^2?, or a(a^2-2a+1)>0?, or a(a-1)^2>0? and ultimately as a cannot equal 1, whether a>0, if we modify the question.
Hence the answer becomes (C) as
condition 1 answers the question 'no' if a=1, but 'yes' when a=2, making it insufficient.
It needs to work with condition 2, so the answer becomes (C).


Once we modify the original condition and the question according to the variable approach method 1, we can solve approximately 30% of DS questions.
User avatar
Manager
Manager
Joined: 29 Nov 2011
Posts: 77
Own Kudos [?]: 77 [0]
Given Kudos: 366
Send PM
Re: Is a + a^(-1) > 2? (1) a > 0 (2) a < 1 [#permalink]
VeritasPrepKarishma wrote:
Bunuel wrote:
Is a + a^(-1) > 2?

(1) a > 0
(2) a < 1

Kudos for a correct solution.


Given expression: \(a + \frac{1}{a}\)

If a is negative, the expression becomes negative and hence less than 2.

Assuming a is positive,
Note that the product of the numbers is a*(1/a) = 1. Given a constant product, the sum of two positive numbers is minimum when they are equal. When a = 1/a, a^2 = 1 and hence a = 1
When a = 1, a + 1/a = 2
So minimum value of a + 1/a is 2. For all other positive values of a, the sum will be more than 2.
Both statements together tell us that a is positive and not 1. So in every case, the sum a + 1/a will be greater than 2. Together they are sufficient to answer the question with "Yes".

Answer (C)



HI Karishma
Please help me here
by solving we get
(a-1)^>0 which we need to prove.

now A is in sufficient as A can be 1.
but in B
put any value less than 1 square will make it >0 . Please let me know what m i missing.
Board of Directors
Joined: 17 Jul 2014
Posts: 2163
Own Kudos [?]: 1180 [0]
Given Kudos: 236
Location: United States (IL)
Concentration: Finance, Economics
GMAT 1: 650 Q49 V30
GPA: 3.92
WE:General Management (Transportation)
Send PM
Re: Is a + a^(-1) > 2? (1) a > 0 (2) a < 1 [#permalink]
Bunuel wrote:
Is a + a^(-1) > 2?

(1) a > 0
(2) a < 1

Kudos for a correct solution.



i will go with C...
a+1/a>2?
1. a>0. if a=0.1, and 1/a=10 then yes, the result is greater than 2. but if a=1, then 1/a=1. 1+1=2 and the answer to the question is no. A/D out.

2. a<1.
a=0.1 1/a=10 => yes
a=-2. 1/a =-1/2 => -2.5<2 so no.
B is out.

1+2.
0<a<1
so a must be a non-integer between 0 and 1. so always will be >2.
for ex. a=0.1, 1/a=10. yes
a=0.25, 1/a=4, yes.

C must be the answer.
User avatar
Non-Human User
Joined: 09 Sep 2013
Posts: 32643
Own Kudos [?]: 821 [0]
Given Kudos: 0
Send PM
Re: Is a + a^(-1) > 2? (1) a > 0 (2) a < 1 [#permalink]
Hello from the GMAT Club BumpBot!

Thanks to another GMAT Club member, I have just discovered this valuable topic, yet it had no discussion for over a year. I am now bumping it up - doing my job. I think you may find it valuable (esp those replies with Kudos).

Want to see all other topics I dig out? Follow me (click follow button on profile). You will receive a summary of all topics I bump in your profile area as well as via email.
GMAT Club Bot
Re: Is a + a^(-1) > 2? (1) a > 0 (2) a < 1 [#permalink]
Moderator:
Math Expert
92901 posts

Powered by phpBB © phpBB Group | Emoji artwork provided by EmojiOne